Revision as of 10:31, 13 September 2013 by Rhea (Talk | contribs)

(diff) ← Older revision | Latest revision (diff) | Newer revision → (diff)


ECE Ph.D. Qualifying Exam

Communication, Networking, Signal and Image Processing (CS)

Question 5: Image Processing

August 2011



Part 1

Jump to Part 1,2


 $ \color{blue}\text{Consider the following discrete space system with input } x(m,n) \text{ and output } y(m,n). $

                $ \color{blue} y(m,n) = \sum_{k=-\infty}^{\infty}{\sum_{l=-\infty}^{\infty}{x(m-k,n-l)h(k,l)}}. $


$ \color{blue} \text{For parts a) and b) let} $
                $ \color{blue} h(m,n)=sinc(mT,nT) $
$ \color{blue} \text{where } T\leq1. $


$ \color{blue} \text{For parts c), d), and e) let} $
                $ \color{blue} h(m,n)=sinc\left( \frac{(n+m)T}{\sqrt[]{2}},\frac{(n-m)T}{\sqrt[]{2}} \right) $
$ \color{blue} \text{where } T\leq1. $


$ \color{blue}\text{a) Calculate the frequency response, }H \left( e^{j\mu},e^{j\nu} \right). $

$ \color{blue}\text{Solution 1:} $

$ \color{green} \text{Recall should be added:} $

$ \color{green} f(am,bn) \overset{DTFT}{\Leftrightarrow } \frac{1}{|a||b|}F(\frac{\mu}{|a|},\frac{\nu}{|b|}) $

$ \color{green} sinc(m,n) \overset{DTFT}{\Leftrightarrow } rect(\mu,\nu) $

$ H(e^{j\mu},e^{j\nu}) = \frac{1}{T^2} rect(\frac{\mu}{T},\frac{\nu}{T}) $


$ \color{blue}\text{Solution 2:} $

$ sinc(m,n) \rightarrow rect(\mu)rect(\nu) $


$ \Rightarrow sinc(mT,nT) \rightarrow \frac{1}{T^2}rect(\frac{\mu}{T})rect(\frac{\nu}{T}) $


$ = H(e^{j\mu},e^{j\nu}) $

$ \color{green} \text{Here, the student uses the Separability property of the sinc and rect functions.} $


$ \color{blue}\text{b) Sketch the frequency response for } |\mu| < 2\pi \text{ and } |\nu| < 2\pi \text{ when } T = \frac{1}{2} $

$ \color{green} \text{Recall should be added:} $

$ \color{green} rect(t) = \left\{\begin{matrix} 1, for |t|\leq \frac{1}{2} \\ 0, otherwise \end{matrix}\right. $


$ {\color{green} \text{Here, the following descriptions should be clarified:} } $

$ {\color{green} \text{Using the separability property for rect function, for } T = \frac{1}{2} { we have:} } $

$ {\color{green} H(e^{j\mu},e^{j\nu}) = \frac{1}{T^2} rect(\frac{\mu}{T},\frac{\nu}{T}) } $

$ {\color{green} = 4 rect(2\mu)rect(2\nu) } $


QE 11 CS5 1 b.png

$ {\color{red} \text{In this sketch it is not mentioned that the gain is } 4. } $


$ \color{blue}\text{Solution 2:} $

$ T = \frac{1}{2}, H(e^{j\mu},e^{j\nu}) = 4rect(2\mu)rect(2\nu) $

QE 11 CS5 1 b sol2.PNG



$ \color{blue}\text{c) Calculate the frequency response, }H \left( e^{j\mu},e^{j\nu} \right). $

$ \color{blue}\text{Solution 1:} $

$ \color{green} \text{Recall should be added:} $

$ \color{green} f \left ( A \begin{bmatrix} m \\ n \end{bmatrix} \right) \overset{DTFT}{\Leftrightarrow } \frac{1}{|A|^{-1}}F([\mu, \nu] A^{-1}) $


$ \color{green} \text{ In this case, A}= \begin{bmatrix} \frac{1}{\sqrt{2}} &\frac{1}{\sqrt{2}} \\ -\frac{1}{\sqrt{2}} &\frac{1}{\sqrt{2}} \end{bmatrix} \text{, hence:} $


$ H(e^{j\mu},e^{j\nu}) = \frac{1}{T^2} rect \left ( \frac{(\mu + \nu)}{\sqrt{2}T},\frac{(\nu - \mu)}{\sqrt{2}T} \right ) $


$ \color{blue}\text{Solution 2:} $

$ \left ( \frac{(n + m)T}{\sqrt{2}},\frac{(n - m)T}{\sqrt{2}} \right) = \begin{bmatrix} \frac{1}{\sqrt{2}} &\frac{1}{\sqrt{2}} \\ -\frac{1}{\sqrt{2}} &\frac{1}{\sqrt{2}} \end{bmatrix} \cdot \begin{pmatrix} mT\\ nT \end{pmatrix} = A \cdot \begin{pmatrix} mT\\ nT \end{pmatrix} $

$ \text{As } |A| = 1, A^{-1} = A^T, sinc \left( A \begin{pmatrix} mT\\ nT \end{pmatrix} \right) \overset{\mathcal{F}}{\rightarrow} F \left( A \begin{pmatrix} \mu\\ \nu \end{pmatrix} \right) $

$ = \frac{1}{T^2} rect \left ( \frac{(\mu + \nu)}{\sqrt{2}T},\frac{(\nu - \mu)}{\sqrt{2}T} \right ) $


$ \color{blue}\text{d) Sketch the frequency response for } |\mu| < 2\pi \text{ and } |\nu| < 2\pi \text{ when } T = \frac{1}{2} $

$ \color{blue}\text{Solution 1:} $

$ \color{green} \text{Recall should be added: Since A is an orthogonal matrix, this transformation is rotationally invariant.} $

$ \color{green} H(e^{j\mu},e^{j\nu}) = \frac{1}{T^2} rect \left ( \frac{(\mu + \nu)}{\sqrt{2}T},\frac{(\nu - \mu)}{\sqrt{2}T} \right ) $

$ \color{green} = 4 rect \left (\sqrt{2} (\mu + \nu),\sqrt{2}(\nu - \mu) \right ) $

$ \color{green} \text{Or} $

$ \color{green} = 4 rect \left (\sqrt{2} (\mu + \nu) \right) rect \left (\sqrt{2}(\nu - \mu) \right ) $


QE 11 CS5 1 d.PNG


$ { \color{red} \text{This sketch is partially correct: The cut-offs should be divided by } 4! } $

$ { \color{red} \text{ Also, it should be mentioned that the gain is} 4. } $


$ \color{blue}\text{Solution 2:} $

$ T = \frac{1}{2}, H(e^{j\mu},e^{j\nu}) = 4rect(\sqrt{2}(\mu + \nu))rect(\sqrt{2}(\nu - \mu)) $

QE 11 CS5 1 d sol2.PNG


$ \color{blue}\text{e) Calculate } y(m,n) \text{ when } x(m,n)=1. $


$ \color{blue}\text{Solution 1:} $


$ Y(e^{j\mu},e^{j\nu}) = \delta(e^{j\mu},e^{j\nu}) \cdot H(e^{j\mu},e^{j\nu}) $

$ = \frac{1}{T^2} rect (0,0) = 4 $

$ \Rightarrow y(m,n) = 4\delta(m,n) $


$ \color{blue}\text{Solution 2:} $

$ y(m,n) = x(m,n) \cdot H(e^{j0},e^{j0}) = 4 $

$ \color{red} \text{The final answer is correct, but the student has skipped some parts of the derivation and the notations do not sound right.} $


"Communication, Networks, Signal, and Image Processing" (CS)- Question 5, August 2011

Go to


Back to ECE Qualifying Exams (QE) page

Alumni Liaison

Abstract algebra continues the conceptual developments of linear algebra, on an even grander scale.

Dr. Paul Garrett